Khi định lý giới hạn trung tâm và định luật về số lượng lớn không đồng ý


19

Đây thực chất là một bản sao của một câu hỏi tôi tìm thấy tại math.se , không nhận được câu trả lời mà tôi hy vọng.

Đặt là một chuỗi các biến ngẫu nhiên độc lập, được phân phối giống hệt nhau, với và .{Xi}iNE[Xi]=1V[Xi]=1

Xem xét đánh giá

limnP(1ni=1nXin)

Biểu thức này phải được thao tác vì cả hai mặt của sự kiện bất bình đẳng có xu hướng vô cùng.

A) ĐĂNG KÝ THỬ

Trước khi xem xét tuyên bố giới hạn, hãy trừ n từ cả hai phía:

limnP(1ni=1nXinnn)=limnP(1ni=1n(Xi1)0)=Φ(0)=12

đẳng thức cuối cùng của CLT, trong đó Φ() là hàm phân phối chuẩn thông thường.

B) ĐA NĂNG

Nhân cả hai bên với 1/n

limnP(1n1ni=1nXi1nn)=limnP(1ni=1nXi1)

=limnP(X¯n1)=limnFX¯n(1)=1

Trong đó FX¯n() là hàm phân phối của trung bình mẫu X¯n , do LLN hội tụ theo xác suất (và cả trong phân phối) cho hằng số 1 , do đó là đẳng thức cuối cùng.

Vì vậy, chúng tôi nhận được kết quả mâu thuẫn. Cái nào đúng? Và tại sao người kia lại sai?


1
@JuhoKokkala Chắc chắn, đây rồi, math.stackexchange.com/q/2830304/87400 Chỉ cần bỏ qua lỗi của OP ở đó.
Alecos Papadopoulos

2
Tôi nghĩ vấn đề nằm ở tuyên bố thứ hai khi gọi LLN
Glen_b -Reinstate Monica

3
Tôi theo bạn cho đến bình đẳng cuối cùng. Rõ ràng là sai, bởi vì chúng tôi mong đợi xấp xỉ cho lớn và do đó giới hạn của nó không nên bằng biện minh dự định của nó là gì? Đây không phải là tuyên bố của bất kỳ phiên bản luật nào với số lượng lớn mà tôi biết. P(X¯n1)1/2n1.
whuber

1
@whuber Giả sử, tất cả xác suất cho mẫu có nghĩa tập trung vào giá trị . Nếu điều này là sai, tôi tin rằng điều quan trọng là lỗi phải được nêu chi tiết trong câu trả lời, đó là mục đích của câu hỏi này. 1
Alecos Papadopoulos

2
Alecos, mối quan tâm của tôi không phải là liệu bước cuối cùng có sai hay không: nó liên quan đến lý do của bạn để thực hiện nó. Không phải đó là tất cả những gì câu hỏi về? Tôi vẫn chưa đọc bất cứ điều gì từ bạn đưa ra những lý do đó và tôi thậm chí sẽ ngần ngại đoán xem chúng có thể là gì. Mặc dù bạn đề cập đến "LLN", tôi tin rằng việc giải quyết vấn đề của bạn có thể nằm trong việc mô tả chính xác những gì bạn hiểu "LLN" để khẳng định.
whuber

Câu trả lời:


15

Lỗi ở đây có khả năng xảy ra trong thực tế sau: hội tụ trong phân phối ngầm giả định rằng hội tụ đến tại các điểm liên tục của . Do phân phối giới hạn là một biến ngẫu nhiên không đổi, nên nó có một bước nhảy không đổi tại , do đó không chính xác khi kết luận rằng CDF hội tụ đến . Fn(x)F(x) F(x)x=1F(x)=1


1
Cách chúng tôi xác định sự hội tụ trong phân phối không loại trừ khả năng hội tụ tại các điểm không liên tục - chỉ là không yêu cầu nó.
Alecos Papadopoulos

1
Nhưng nếu sự hội tụ trong phân phối không yêu cầu hội tụ đến F ( 1 ) , thì đẳng thức cuối cùng trong câu hỏi dựa trên điều gì? Fn(1)F(1)
Juho Kokkala

1
@Juho Nó không dựa trên bất cứ điều gì - đó là mấu chốt của vấn đề. Không có định lý nào cho phép người ta lập phương trình cuối cùng trong câu hỏi.
whuber

1
@AlecosPapadopoulos: Tôi chưa bao giờ nói rằng nó không loại trừ khả năng. Tôi đang ngầm nói rằng bạn cần biện minh cho sự bình đẳng cuối cùng vượt ra ngoài những gì được trao cho bạn từ sự hội tụ trong phân phối. Ví dụ nếu là Bernoulli, thì đó là sự thật. Xn
Alex R.

11

Cho iid các biến ngẫu nhiên với E [ X i ] = var ( X i ) = 1 xác định Z nXiE[Xi]=var(Xi)=1 Bây giờ, CLT nói rằng đối với mỗicố địnhsố thựcz,limnFZn(z)=Φ(z-1). OP áp dụng CLT để đánh giá limnP(Zn1

Zn=1ni=1nXi,Yn=1ni=1nXi.
zlimnFZn(z)=Φ(z1)
limnP(Zn1n)=Φ(0)=12.

Như câu trả lời khác cũng như một số các ý kiến về câu hỏi của OP đã chỉ ra, đó là đánh giá của OP của đó là nghi ngờ. Hãy xem xét trường hợp đặc biệt khi iid X i là các biến ngẫu nhiên rời rạc lấy các giá trị 02 với xác suất 1 bằng nhaulimnP(Yn1)Xi02 . Bây giờ, n i = 1 Xtôicó thể nhậntấtcả các giá trị nguyên chẵn trong[0,2n]và vì vậy khinlà số lẻ, n i = 1 Xtôikhông thể nhận giá trịnvà do đóYn=112i=1nXi[0,2n]ni=1nXinkhông thể nhận giá trị1. Hơn nữa, vì sự phân bố củaYnlà đối xứng về1, ta có P(Yn1)=FYn(1)có giá trị1Yn=1ni=1nXi 1Yn1P(Yn1)=FYn(1) bất cứ khi nàonlà số lẻ. Như vậy,chuỗicác số P(Y11),P(Y21),...,P(Yn1),... chứa cácdãyP(Y11),P(Y31),Lọ,P(Y12n

P(Y11),P(Y21),,P(Yn1),
trong đó tất cả các điều khoản có giá trị 1
P(Y11),P(Y31),,P(Y2k11),
. Mặt khác, cácdãyP(Y21),P(Y41),...,P(Y2k1),... đượchội tụđể1. Do đó,limnP(Yn1)không tồn tại và tuyên bố của hội tụ củaP(Yn112
P(Y21),P(Y41),,P(Y2k1),
1limnP(Yn1) đến 1 phải được xem với rất nhiều sự nghi ngờ.P(Yn1)

8

Kết quả đầu tiên của bạn là chính xác. Lỗi của bạn xảy ra trong phần thứ hai, trong câu lệnh sai sau đây:

limnFX¯n(1)=1.

Tuyên bố này là sai (phía bên tay phải là ) và nó không tuân theoluật số lượng lớnnhư đã khẳng định. Luật yếu của số lượng lớn (mà bạn viện dẫn) nói rằng:12

limnP(|X¯n1|ε)=1for all ε>0.

Đối với tất cả điều kiện | ˉ X n - 1 | ε nhịp một số giá trị nơi ˉ X n1 và một số giá trị nơi ˉ X n > 1 . Do đó, nó không làm theo từ LLN rằng lim n P ( ˉ X n1 ) = 1 .ε>0|X¯n1|εX¯n1X¯n>1limnP(X¯n1)=1


1
Kết quả (thực sự sai lầm) xuất phát từ hàm ý "sự hội tụ trong xác suất ngụ ý sự hội tụ trong phân phối". Câu hỏi không nêu rõ rằng khẳng định đến trực tiếp từ LLN.
Alecos Papadopoulos

@AlecosPapadopoulos: Sự hội tụ trong xác suất không bao hàm sự hội tụ trong phân phối. Một lần nữa, sự hội tụ trong phân phối chỉ được yêu cầu tại các điểm liên tục. Nhưng, có lẽ bạn có nghĩa là sự hội tụ trong xác suất không bao hàm sự hội tụ theo chiều hướng phân phối.
Alex R.

@AlexR. Tôi không chắc chắn sự phản đối của bạn nằm ở đâu. Tôi tin rằng vấn đề này được đề cập trong câu trả lời của riêng tôi.
Alecos Papadopoulos

3

Sự hội tụ trong xác suất ngụ ý sự hội tụ trong phân phối. Nhưng ... phân phối gì? Nếu phân phối giới hạn có gián đoạn bước nhảy thì các giới hạn trở nên mơ hồ (vì có thể có nhiều giá trị tại điểm gián đoạn).

Trong đó là hàm phân phối của giá trị trung bình mẫu ˉ X n , do LLN hội tụ theo xác suất (và cả trong phân phối) cho hằng số 1 ,FX¯n()X¯n1

This is not right, and it is also easy to show that it can not be right (different from the disagreement between CLT and LLN). The limiting distribution (which can be seen as the limit for a sequence of normal distributed variables) should be:

FX¯(x)={0for x<10.5for x=11for x>1

ϵ>0x|FX¯n(x)FX¯(x)|<ϵnFX¯(1)=1 instead of FX¯(1)=0.5


Limit of a normal distribution

It may be helpful to explicitly write out the sum used to invoke the law of large numbers.

X¯n=1ni=1nXiN(1,1n)

The limit n for X^n is actually equivalent to the Dirac Delta function when it is represented as the limit of the normal distribution with the variance going to zero.

Using that expression it is more easy to see what is going on under the hood, rather than using the ready-made laws of the CLT an LLN which obscure the reasoning behind the laws.


Convergence in probability

The law of large numbers gives you 'convergence in probability'

limnP(|X¯n1|>ϵ)=0

with ϵ>0

An equivalent statement could be made for the central limit theorem with limnP(|1n(Xi1)|>ϵn)=0

It is wrong to state that this implies

limnP(|X¯n1|>0)=0

It is less nice that this question is cross-posted so early (confusing, yet interesting to see the different discussions/approaches math vs stats, so not that too bad). The answer by Michael Hardy on the math stackexchange deals with it very effectively in terms of the strong law of large numbers (the same principle as the accepted answer from drhab in the cross posted question and Dilip here). We are almost sure that a sequence X¯1,X¯2,X¯3,...X¯n converges to 1, but this does not mean that limnP(X¯n=1) will be equal to 1 (or it may not even exist as Dilip shows). The dice example in the comments by Tomasz shows this very nicely from a different angle (instead of the limit not existing, the limit goes to zero). The mean of a sequence of dice rolls will converge to the mean of the dice but the probability to be equal to this goes to zero.


Heaviside step function and Dirac delta function

The CDF of X¯n is the following:

FX¯n(x)=12(1+erfx12/n)

with, if you like, limnFX¯n(1)=0.5 (related to the Heaviside step function, the integral of the Dirac delta function when viewed as the limit of normal distribution).


I believe that this view intuitively resolves your question regarding 'show that it is wrong' or at least it shows that the question about understanding the cause of this disagreement of CLT and LLN is equivalent to the question of understanding the integral of the Dirac delta function or a sequence of normal distributions with variance decreasing to zero.


2
Your limiting distribution is in fact not a distribution at all. A CDF must be right continuous, whereas it clearly is not at x=1/2.
Alex R.

The right continuity seems to be necessary such that for every a we have limnFX(a+1n)=FX(a) as the events Xa+1n are nested we should have
limnFX(a+1n)=limnP(Xa+1n)=P(limnXa+1n)=P(Xa)=FX(a)
but is this true for our case and where is the catch? Is this right continuity necessary based on probability axioms or is it just a convention such that the CDF works for most common cases?
Sextus Empiricus

@Martin Weterings: This is precisely where it comes from. Any valid measure P must satisfy these monotonicity results. They are a consequence of the boundedness of P along with countable additivity. More generally, a function F(x) is a CDF (i.e. corresponds to some distribution P via F(b)F(a)=P(a<Xb) iff F is right-continuous, along with being monotonic, and having left limit 0, right limit 1.
Alex R.

2

I believe it should be clear by now that "the CLT approach" gives the right answer.

Let's pinpoint exactly where the "LLN approach" goes wrong.

Starting with the finite statements, it is clear then that we can equivalently either subtract n from both sides, or multliply both sides by 1/n. We get

P(1ni=1nXin)=P(1ni=1n(Xi1)0)=P(1ni=1nXi1)

So if the limit exists, it will be identical. Setting Zn=1ni=1n(Xi1), we have, using distribution functions

P(1ni=1nXin)=FZn(0)=FX¯n(1)

...and it is true that limnFZn(0)=Φ(0)=1/2.

The thinking in the "LLN approach" goes as follows: "We know from the LLN that X¯n converges in probabililty to a constant. And we also know that "convergence in probability implies convergence in distribution". So, X¯n converges in distribution to a constant". Up to here we are correct.
Then we state: "therefore, limiting probabilities for X¯n are given by the distribution function of the constant at 1 random variable",

F1(x)={1x10x<1F1(1)=1

... so limnFX¯n(1)=F1(1)=1...

...and we just made our mistake. Why? Because, as @AlexR. answer noted, "convergence in distribution" covers only the points of continuity of the limiting distribution function. And 1 is a point of discontinuity for F1. This means that limnFX¯n(1) may be equal to F1(1) but it may be not, without negating the "convergence in distribution to a constant" implication of the LLN.

And since from the CLT approach we know what the value of the limit must be (1/2). I do not know of a way to prove directly that limnFX¯n(1)=1/2.

Did we learn anything new?

I did. The LLN asserts that

limnP(|X¯n1|ε)=1for all ε>0

limn[P(1ε<X¯n1)+P(1<X¯n1+ε)]=1

limn[P(X¯n1)+P(1<X¯n1+ε)]=1

The LLN does not say how is the probability allocated in the (1ε,1+ε) interval. What I learned is that, in this class of convergence results, the probability is at the limit allocated equally on the two sides of the centerpoint of the collapsing interval.

The general statement here is, assume

Xnpθ,h(n)(Xnθ)dD(0,V)

where D is some rv with distribution function FD. Then

limnP[Xnθ]=limnP[h(n)(Xnθ)0]=FD(0)

...which may not be equal to Fθ(0) (the distribution function of the constant rv).

Also, this is a strong example that, when the distribution function of the limiting random variable has discontinuities, then "convergence in distribution to a random variable" may describe a situation where "the limiting distribution" may disagree with the "distribution of the limiting random variable" at the discontinuity points. Strictly speaking, the limiting distribution for the continuity points is that of the constant random variable. For the discontinuity points we may be able to calculate the limiting probability, as "separate" entities.


The 'lesson learned' perspective is interesting, and this is a good, not too difficult, example for didactic application. Although I wonder what kind of (direct) practical application this thinking about the infinite has, because eventually in practice n
Sextus Empiricus

@MartijnWeterings Martijn, the motivation here was certainly educational, a) as an alert to discontinuities even in such a "flat" situation as the convergence to a constant, and so also in general (they destroy uniform convergence for example), and b) a result on how the probability mass is allocated becomes interesting when the sequence that converges in probabilty to a constant, still has a non-zero variance.
Alecos Papadopoulos

We could say that CLT let's as say something about convergence to a limiting normal distributed variable (thus being able to express such things as F(x)), but LLN only allows us to say that, by increasing the sample size, we get closer to the true mean, but this does not say that we get, with higher probability, 'exactly equal to the sample mean'. LLN means that the sample mean gets closer and closer to a limiting value but not (with higher probability) equal to it. LLN says nothing about F(x)
Sextus Empiricus

The original thoughts around the LLN where actually opposite (see the reasoning of Arbuthnot stats.stackexchange.com/questions/343268). "It is visible from what has been said, that with a very great Number of Dice, A’s Lot would become very small... there would be but a small part of all the possible Chances, for its happening at any assignable time, that an equal Number of Males and Females should be born."
Sextus Empiricus
Khi sử dụng trang web của chúng tôi, bạn xác nhận rằng bạn đã đọc và hiểu Chính sách cookieChính sách bảo mật của chúng tôi.
Licensed under cc by-sa 3.0 with attribution required.